matheraum.de
Raum für Mathematik
Offene Informations- und Nachhilfegemeinschaft

Für Schüler, Studenten, Lehrer, Mathematik-Interessierte.
Hallo Gast!einloggen | registrieren ]
Startseite · Forum · Wissen · Kurse · Mitglieder · Team · Impressum
Forenbaum
^ Forenbaum
Status Schulmathe
  Status Primarstufe
  Status Mathe Klassen 5-7
  Status Mathe Klassen 8-10
  Status Oberstufenmathe
    Status Schul-Analysis
    Status Lin. Algebra/Vektor
    Status Stochastik
    Status Abivorbereitung
  Status Mathe-Wettbewerbe
    Status Bundeswettb. Mathe
    Status Deutsche MO
    Status Internationale MO
    Status MO andere Länder
    Status Känguru
  Status Sonstiges

Gezeigt werden alle Foren bis zur Tiefe 2

Navigation
 Startseite...
 Neuerdings beta neu
 Forum...
 vorwissen...
 vorkurse...
 Werkzeuge...
 Nachhilfevermittlung beta...
 Online-Spiele beta
 Suchen
 Verein...
 Impressum
Das Projekt
Server und Internetanbindung werden durch Spenden finanziert.
Organisiert wird das Projekt von unserem Koordinatorenteam.
Hunderte Mitglieder helfen ehrenamtlich in unseren moderierten Foren.
Anbieter der Seite ist der gemeinnützige Verein "Vorhilfe.de e.V.".
Partnerseiten
Weitere Fächer:

Open Source FunktionenplotterFunkyPlot: Kostenloser und quelloffener Funktionenplotter für Linux und andere Betriebssysteme
StartseiteMatheForenZahlentheoriebinomischer Lehrsatz
Foren für weitere Schulfächer findest Du auf www.vorhilfe.de z.B. Philosophie • Religion • Kunst • Musik • Sport • Pädagogik
Forum "Zahlentheorie" - binomischer Lehrsatz
binomischer Lehrsatz < Zahlentheorie < Algebra+Zahlentheo. < Hochschule < Mathe < Vorhilfe
Ansicht: [ geschachtelt ] | ^ Forum "Zahlentheorie"  | ^^ Alle Foren  | ^ Forenbaum  | Materialien

binomischer Lehrsatz: Primzahlkongruenz
Status: (Frage) beantwortet Status 
Datum: 22:15 Do 09.06.2011
Autor: clemenum

Aufgabe
Man zeige: [mm] $\forall [/mm] a,b [mm] \in \mathbb{Z}, \forall p\in \mathbb{P}: (a+b)^p \equiv a^p+b^p(p)$ [/mm]

Offensichtlich genügt es [mm] $p|{p\choose i} \forall 0\le [/mm] i [mm] \le [/mm] p-1$ zu zeigen.
Es gilt ja:
[mm] $(a+b)^p [/mm] = [mm] \sum_{i=0}^p {n\choose i} a^i b^{p-i}$ [/mm]
Es ist ja nach Definition [mm] {n\choose i} [/mm] = [mm] \frac{p(p-1)\cdot\ldots\cdot(p-i+1)}{i(i-1)\ldots2\cdot1}\in \mathbb{Z},$ [/mm] womit ersichtlich der Zähler durch p teilbar ist.
Es bleibt also "nur noch" zu zeigen, dass [mm] $(p-1)(p-2)\ldots(p-i+1)$ [/mm] ein ganzzahliges Vielfaches von $i!$ ist.
Nun, i bewegt sich ja zwischen [mm] $0\le [/mm] i [mm] \le [/mm] p-1$. Ich sehe hier für die meisten Fällen einen Bruch vor mir, dessen Zähler kleiner als der Nenner ist, womit die Aussage nicht bewiesen sondern widerlegt wäre! :-/
Setze ich etwa $i = p-1 $, so erhalte ich [mm] $\frac{p}{(p-i)(p-i-1)\ldots\cdot2\cdot1}$ [/mm] und da kann offensichtlich etwas nicht stimmen!

Wie könnte ich vorgehen, damit ich die Aussage doch beweisen kann?

        
Bezug
binomischer Lehrsatz: Antwort
Status: (Antwort) fertig Status 
Datum: 22:35 Do 09.06.2011
Autor: wieschoo


> Man zeige: [mm]\forall a,b \in \mathbb{Z}, \forall p\in \mathbb{P}: (a+b)^p \equiv a^p+b^p(p)[/mm]
>  
> Offensichtlich genügt es [mm]p|{p\choose i} \forall 0\le i \le p-1[/mm]
> zu zeigen.
> Es gilt ja:
> [mm](a+b)^p = \sum_{i=0}^p {n\choose i} a^i b^{p-i}[/mm]
> Es ist ja nach Definition [mm]{n\choose i}[/mm] =
> [mm]\frac{p(p-1)\cdot\ldots\cdot(p-i+1)}{i(i-1)\ldots2\cdot1}\in \mathbb{Z},$[/mm]

Genau!
[mm]{{p \choose n}} =\frac{\prod_{j=0}^{n-1}(p-j)}{n!}[/mm]

Hier kannst du doch direkt begründen. $p>n$. Somit ist doch p größer als jede Primzahl, die n! teilt.

Was sagt dir:

[mm]{{p \choose n}}n! =\prod_{j=0}^{n-1}(p-j)[/mm] und [mm]p | \prod_{j=0}^{n-1}(p-j)[/mm]

Was weißt du über das "Verhältnis" von n! und p?


> womit ersichtlich der Zähler durch p teilbar ist.
> Es bleibt also "nur noch" zu zeigen, dass
> [mm](p-1)(p-2)\ldots(p-i+1)[/mm] ein ganzzahliges Vielfaches von [mm]i![/mm]
> ist.
> Nun, i bewegt sich ja zwischen [mm]0\le i \le p-1[/mm]. Ich sehe
> hier für die meisten Fällen einen Bruch vor mir, dessen
> Zähler kleiner als der Nenner ist, womit die Aussage nicht
> bewiesen sondern widerlegt wäre! :-/
> Setze ich etwa [mm]i = p-1 [/mm], so erhalte ich
> [mm]\frac{p}{(p-i)(p-i-1)\ldots\cdot2\cdot1}[/mm] und da kann
> offensichtlich etwas nicht stimmen!
>
> Wie könnte ich vorgehen, damit ich die Aussage doch
> beweisen kann?





Bezug
                
Bezug
binomischer Lehrsatz: Frage (beantwortet)
Status: (Frage) beantwortet Status 
Datum: 23:19 Do 09.06.2011
Autor: clemenum

Also, aus deinen Aussagen folgt unmittelbar:
[mm] $p|{p\choose n}$ $\vee$ [/mm] $p|n!$. Da du (und nun auch ich) festgstellt hast, dass für [mm] $p>p_i: p_i|n! [/mm] $ [mm] $\forall [/mm] i $ ist nun offensichtlich, dass $p$ nicht $n!$ teilen kann, womit zwangsläufig nur mehr die Möglichkeit, dass [mm] $p|{p\choose n}$ [/mm] gilt, übrigbleibt.

Hast du dir die Argumentation so vorgestellt?

Bezug
                        
Bezug
binomischer Lehrsatz: Antwort
Status: (Antwort) fertig Status 
Datum: 23:34 Do 09.06.2011
Autor: reverend

Hallo clemenum,

> Also, aus deinen Aussagen folgt unmittelbar:
>  [mm]p|{p\choose n}[/mm] [mm]\vee[/mm] [mm]p|n![/mm]. Da du (und nun auch ich)
> festgstellt hast, dass für [mm]p>p_i: p_i|n![/mm] [mm]\forall i[/mm] ist nun
> offensichtlich, dass [mm]p[/mm] nicht [mm]n![/mm] teilen kann, womit
> zwangsläufig nur mehr die Möglichkeit, dass [mm]p|{p\choose n}[/mm]
> gilt, übrigbleibt.
>
> Hast du dir die Argumentation so vorgestellt?  

Du machst es Dir zu kompliziert.
Du kannst mit der vorliegenden Formel zur Berechnung der Binomialkoeffizienten doch ganz direkt zeigen, dass [mm] p|\vektor{p\\i}\ \forall [/mm] 0<i<p. Außerdem gilt [mm] \vektor{p\\0}=\vektor{p\\p}=1. [/mm]

Und damit ist die Aufgabe doch schon gelöst.
Schau nochmal genau hin und überleg Dir, welche Faktoren das Zählerprodukt beinhaltet.

Im übrigen kannst Du die Aufgabe auch schnell auf anderem Weg lösen, wenn Du den Satz von Fermat benutzen darfst (bzw. den Satz von Euler-Fermat).

Grüße
reverend




Bezug
                                
Bezug
binomischer Lehrsatz: Nachfrage
Status: (Frage) beantwortet Status 
Datum: 14:23 Fr 10.06.2011
Autor: wieschoo

Deine Argumentation ist mir leider nicht klar.
Wenn ich das nachvollziehe, dann kann ich doch auch schließen: "5 teilt 2", da ja [mm]2=\frac{2*5}{5}[/mm] ist und man aus dem Zähler die 5 herausziehen kann.

Also irgendwie muss man doch auch begründen, dass die Zähler nicht dagegen schießt.

Bezug
                                        
Bezug
binomischer Lehrsatz: Antwort
Status: (Antwort) fertig Status 
Datum: 23:46 Fr 10.06.2011
Autor: reverend

Hallo wieschoo,

Beweisweg A:
1) Jedes [mm] p\in\IP [/mm] besitzt nur die trivialen Teiler 1 und p. Insbesondere gilt also [mm] k\not|p\ \forall 1 2) Es ist [mm] \vektor{p\\k}=\bruch{p!}{k!(p-k)!}. [/mm] Sei [mm] 1\le k\le{p}. [/mm] Dann sind alle Faktoren des Nenners <p und teilen p daher nicht. Mithin gilt dann [mm] p|\vektor{p\\k} [/mm]
3) Also gilt [mm] (a+b)^p=a^p+b^p+\summe_{k=1}^{p-1}\vektor{p\\k}a^k b^{p-k}\equiv a^p+b^p \mod{p} [/mm]

Beweisweg B:
1) Für [mm] p\in\IP [/mm] gilt [mm] n^p\equiv n\mod{p}, [/mm] für [mm] n\not\equiv 0\mod{p} [/mm] folgt dies aus dem Satz von Fermat, für [mm] n\equiv 0\mod{p} [/mm] ist es trivial.
2) Also gilt [mm] (a+b)^p\equiv a+b\equiv a^p+b^p\mod{p}. [/mm]

Grüße
reverend


Bezug
                                                
Bezug
binomischer Lehrsatz: Mitteilung
Status: (Mitteilung) Reaktion unnötig Status 
Datum: 11:05 Sa 11.06.2011
Autor: wieschoo

Danke dir. Die Idee B kannte ich noch nicht.


Bezug
                        
Bezug
binomischer Lehrsatz: Mitteilung
Status: (Mitteilung) Reaktion unnötig Status 
Datum: 14:24 Fr 10.06.2011
Autor: wieschoo

So hatte ich es mir gedacht.

Falls es doch einfacher geht, dann wäre ich auch daran interessiert.

Bezug
Ansicht: [ geschachtelt ] | ^ Forum "Zahlentheorie"  | ^^ Alle Foren  | ^ Forenbaum  | Materialien


^ Seitenanfang ^
www.schulmatheforum.de
[ Startseite | Forum | Wissen | Kurse | Mitglieder | Team | Impressum ]